Sample Test HS347

अब Quizwiz के साथ अपने होमवर्क और परीक्षाओं को एस करें!

Which of the following statements concerning the transfer of a business interest to a grantor-retained annuity trust (GRAT) by a senior generation family member is correct? (A) The GRAT is particularly successful as an estate-freezing technique if the property appreciates significantly after the transfer to the GRAT. (B) A GRAT that provides a lifetime annuity to the grantor of the trust is a useful estate reduction planning technique. (C) A GRAT is useful in shifting income from the business interest to family members in the lower income tax bracket. (D) A GRAT provides the grantor with the actual income earned by the trust for a term of years.

(A). (B) is incorrect because a GRAT providing a lifetime annuity will cause inclusion of the principal in the grantor's gross estate because the grantor will be treated as making a transfer but retaining enjoyment for life. (C) is incorrect because the GRAT is typically designed as a grantor or defective trust and the grantor is taxed on the income until the trust terminates. (D) is incorrect because the GRAT must provide an annuity or a fixed sum of money and the retention of the actual income earned by the trustee would not be an effective gift and estate tax planning technique.

Which of the following statements concerning the continuity problems for a closely held business is correct? (A) An S corporation continues to exist at the death of a shareholder who was providing substantial services for the corporation. (B) A general partnership has continuity of life as an entity beyond the death of any general partner. (C) A limited partnership terminates by operation of law at the death or disability of a limited partner. (D) An LLC terminates by operation of law at the death or disability of a member.

(A). (B) is incorrect because in some states a general partnership terminates by operation of law at the death of a general partner unless continuation is provided for in a binding partnership agreement. (C) is incorrect because the entry and exit of limited partners has no effect on the legal status of a limited partnership. (D) is incorrect because the legal status of an LLC is provided for by state law and its operating agreement and the death or disability of a member will generally have no effect on the legal status of the LLC.

Mr. and Mrs. Durrant have a special needs child. In their wills, they create a third-party supplemental needs trust to provide for the quality of life for their child. The trustee is directed to make distributions for the benefit of the child that supplement but do not supplant benefits available from government programs. After the trust is funded, which of the following distributions will be treated as income and potentially reduce the benefits available under SSI and Medicaid? (A) a payment of cash to the child to be used for the monthly food bill (B) a payment to compensate a caregiver of the child (C) a purchase of kitchen appliances for the child (D) a payment of the child's cell phone bill

(A). (B), (C) and (D) are trust benefits for supplemental items, not for the food and shelter of the child. These payments do not count as income under the SSI rules.

All of the following are examples of blended families EXCEPT: (A) a grandmother raising her two grandchildren following the death of their parents (B) a remarried mother of one raising her daughter with her new husband (C) two married adults living together with their children from previous marriages (D) a father with a child from a previous nonmarriage relationship married to a woman without children of her own and raising his child together

(A). A blended family, also known as a stepfamily, is one in which children from a previous marriage or relationship are raised in a household with a stepparent.

All of the following statements concerning the computation of federal estate tax are correct EXCEPT: (A) Deduction and credit are interchangeable terms for the purpose of federal estate tax computations. (B) The estate and gift tax systems are used for transfers made during a lifetime and at death. (C) The credit for tax on prior transfers avoids double taxation on the same assets transferred through two estates within a relatively short time period. (D) Gift taxes paid on post-1976 gifts made by a decedent within 3 years of death become part of the gross estate.

(A). A deduction reduces the gross estate to determine the tax base, while a credit reduces the tax liability on a dollar-per-dollar basis.

All of the following statements regarding considerations in developing a tax-effective withdrawal strategy from a taxable, tax-deferred, and tax-exempt account are correct EXCEPT: (A) A client should always take a distribution from a tax-deferred account when the distribution would be taxed at capital gains rates. (B) A tax-exempt account is not subject to federal income tax if the distribution is a qualified distribution. (C) Taxable income could impact the taxation of social security benefits (D) The federal capital gains rate does not apply to a distribution from a tax-deferred account.

(A). A distribution from a tax-deferred account is taxed at the federal ordinary income tax rates.

All of the following statements regarding retirement income sources are correct EXCEPT: (A) The federal Social Security benefit program provides retirement benefits in the form of a lump sum only to an individual who meets the retirement age and covered employment requirements. (B) An individual who desires to know what her Social Security retirement benefits may be should contact the Social Security Administration or establish an account at the SSA website. (C) An employer-sponsored qualified defined-benefit plan pays a benefit in the form of an annuity as a normal form of benefit and could provide an election to take another form of payment, subject to spousal consent if the participant is married. (D) An individual who desires to know what her employer-provided retirement benefits may be should contact the plan administrator of the plan.

(A). A lump sum form of benefit is not a payment option under the federal Social Security benefit program.

Which of the following statements concerning Social Security benefits in the divorce process is (are) correct? I. Early retirement benefits can begin at age 62 for a divorced spouse from the ex-spouse's benefit as long as the ex-spouse is of retirement age. II. The ex of a surviving spouse cannot collect a full retirement benefit from the account of the ex if they were married for 10 years. (A) I only (B) II only (C) Both I and II (D) Neither I nor II

(A). A reduced benefit can be received from Social Security by an ex-spouse as early as age 62 provided the spouse whose benefit is being used for the calculation is of retirement age.

Which statement, if said by a financial advisor to a client, is most likely to help develop rapport? (A) "You've got a unique situation. Let's work together to prioritize your goals." (B) "If you buy this policy, it will solve all of your needs." (C) "I've seen this problem a million times and will tell you exactly what to do." (D) "Our approach is 'tried and true'—if you don't like it, you're welcome to find another advisor."

(A). Advisors who emphasize their clients' unique situation and their willingness to work together are most likely to develop rapport. Advisors are less likely to develop rapport if they seem like they are selling something (B). Likewise, advisors who push a specific approach—even if grounded in their experience or expertise—may alienate clients and fail to develop rapport.

All of the following statements concerning operating a closely held business in the form of an S corporation are correct EXCEPT: (A) An S corporation is permitted to operate without the formalities of a board of directors. (B) The shareholders of the S corporation are taxed on all business income for the year. (C) Only a domestic corporation can make the S election. (D) An S corporation's ownership is generally limited to individuals and certain eligible trusts.

(A). An S corporation is a normal corporation for all purposes except federal income taxes. State law imposed formalities of operations must be followed.

All of the following statements are true regarding ABLE accounts EXCEPT: (A) An individual can have more than one ABLE account as long as the accounts are in different states. (B) Contributions to an ABLE account may be made by the individual with special needs, family members of the individual, or friends of the individual. (C) Funds in an ABLE account can only be used for expenses related to an individual's disabilities that are not covered by government programs. (D) Investments in an ABLE account can be adjusted up to two times per year.

(A). An individual is only eligible for one ABLE account.

Which of the following statements concerning child support is (are) correct? I. Child support may be modified by a court order as a result of job loss or other extenuating factors. II. Child support is tax decutible to the payer. (A) I only (B) II only (C) Both I and II (D) Neither I nor II

(A). Child support payments are not tax deductible to the payer, nor are they considered taxable income to the payee.

All of the following statements concerning qualified retirement plans are correct EXCEPT: (A) The amount of the value for a defined-contribution plan is known in advance when planning for retirement at a specified date. (B) The longer the years of service, the greater the contributions will be under a defined-benefit plan. (C) Defined-contribution plans are called participant-directed plans since they allow an employee to allocate investments among a variety of options. (D) Under the Retirement Equity Act of 1984, a worker may not opt out of survivor benefits without the knowledge and agreement of the spouse.

(A). Defined-contribution plans have the contribution going into the account defined or known in advance, but there is no way to value what the account will grow to during the growth or distribution periods.

All of the following concepts are part of behavioral finance research EXCEPT: (A) encroachment (B) sunk cost (C) availability (D) confirmation

(A). Encroachment is not part of behavioral finance research.

Which of the following situations are examples of reference dependence? I. William invests $1,000 in Schwarma Pharmaceuticals, which grows to $3,500 before dropping back down to $2,000, at which point William sells the stock. William is upset that he didn't get out when the investment was at its peak of $3,500. II. Jennifer is very pleased that she inherited $10,000 from her grandmother. (A) I only (B) II only (C) Both I and II (D) Neither I nor II

(A). I is correct because William creates a reference point at $3,500, so he is disappointed that he "lost" $1,500 - even though he realized a gain of $1,000 above his original investment. Reference dependence describes a situation where an individual sets a "reference" point for decisions, opinions, and experiences. II is incorrect because although Jennifer is pleased that she inherited $10,000, it is not referenced to any specific amount.

What is an eligible rollover distribution? I. A distribution from one tax-advantaged retirement plan to another tax-advantaged retirement plan to maintain the same tax treatment II. A hardship withdrawal from a 401(k) plan (A) I only (B) II only (C) Both I and II (D) Neither I nor II

(A). I is correct. II is incorrect because a hardship withdrawal does not qualify as an eligible rollover distribution.

Each of the following are factors internal to the decision maker in suboptimal decision making EXCEPT: (A) information (B) bias (C) framing (D) heuristics

(A). Information is external to the decision maker. (B), (C), and (D) are internal, and often result in suboptimization.

All of the following transfers could utilize the annual exclusion EXCEPT: (A) A donor retains a lifetime interest to live in the family home, with the home passing to the donor's children at death. (B) A donor retains a reversionary interest in a trust, with the income rights going to a qualified charity. (C) A donor makes an outright gift of $50,000 to a nonprofit university. (D) A donor makes a gift of the family's beach home to the donor's children for life but dictates that at the children's deaths, the home would be passed to her alma matter.

(A). It is the only answer choice for which a present interest is not gifted; therefore the annual exclusion is not allowed.

In accordance with the "Tax Benefit Rule," if a client, Karla Edwards, took a deduction for medical expenses paid in the amount of $20,000 in 2018 and received a reimbursement from her health insurance provider in the amount of $8,000 in 2019, how much of this reimbursement should Karla include in her 2019 income, assuming her itemized deductions for 2018 equaled $21,000? (A) $8,000 (B) $9,000 (C) $12,000 (D) $20,000

(A). Karla must include in income the reimbursement to the extent of the lesser of the prior year's deduction ($20,000), or the amount by which the prior year's itemized deductions exceeded the standard deduction ($21,000 - $12,000 = $9,000) or the amount of the reimbursement ($8,000).

It is easier to assess whether or not a goal is attainable if it is: (A) quantitative and specific (B) qualitative and specific (C) quantitative and general (D) qualitative and general

(A). Most goals begin as qualitative and general. Advisors should help their clients refine them into quantitative, specific goals so as to better assess whether or not these goals are attainable.

Each of the following is an example of decision biases/heuristics that differentiates behavioral finance EXCEPT: (A) preference for delayed gratification (B) making a decision based on one's own geographical region (C) holding onto a portfolio of undiversified assets due to an illusion of control (D) seeking information to support a decision or belief

(A). Preference for delayed gratification is not a behavioral heuristic. All the other choices illustrate biases in behavioral finance identified by researchers.

All of the following statements are true of a qualified domestic relations order (QDRO) EXCEPT: (A) Because obtaining and executing a QDRO is so complicated, it is rarely utilized by divorcing couples. (B) It provides an equitable amount of dollars for each spouse at retirement. (C) It allows for the divisions of qualified retirement plans at divorce. (D) A QDRO is a binding legal document that must be prepared by an attorney.

(A). QDROs provide greater simplicity, rather than complexity, to the divorce process.

An irrevocable life insurance trust (ILIT) might be implemented for a closely held business owner for which of the following reasons? I. to provide income to the surviving spouse and an inheritance to family heirs who will not remain active in the business II. to provide the grantor with the ability to change the beneficiary designation for the insurance proceeds once the successors to the closely held business have been determined (A) I only (B) II only (C) Both I and II (D) Neither I nor II

(A). Statement II is incorrect because the ILIT terms are finalized once the trust is created and funded. The grantor no longer has control of the policy and is not allowed to amend the terms of the trust.

Which of the following result(s) from an installment sale of a closely held business interest to family successors? I. This is a good estate-freezing technique because the business interest is removed from the seller's gross estate. II. The installment note is valued at zero for the seller's federal estate tax purposes, even if the seller dies before all the installments are paid. (A) I only (B) II only (C) Both I and II (D) Neither I nor II

(A). Statement II is incorrect because the installment note is an asset of the estate and its value is included in the gross estate unless the seller dies after the note has been satisfied.

Which of the following statements concerning the federal taxation of an LLC is (are) correct? I. The members of an LLC will generally choose to be taxed as a partnership. II. The members of an LLC are treated as statutory employees for federal tax purposes if they provide services to the LLC. (A) I only (B) II only (C) Both I and II (D) Neither I nor II

(A). Statement II is incorrect because the members of an LLC are treated as self-employed. The only business owners that are treated as statutory employees are shareholder-employees of a regular corporation and shareholder-employees of an S corporation who own less than 2 percent of the stock.

Four individuals would like to combine services and resources to operate a closely held business. All will be active in the company activities. They would like to reinvest earnings in the business and defer income from current taxation on their individual federal income tax returns. Which of the following entities would be appropriate for these circumstances? (A) corporation (B) S corporation (C) general partnership (D) LLC

(A). Statements (B), (C), and (D) are incorrect because these entities would all cause the owners of the business to be taxed currently on business income as they are "pass through" entities.

The CPI-U might be most useful to a financial planner for estimating which of the following? (A) Growth in the cost of goods for a resident of a major metropolitan area (B) Medical cost inflation for a recently retired client (C) Undergrad tuition increases at 4-year public universities (D) Expected long-term care costs for a 55-year-old client

(A). The CPI-U is a general measure of inflation for urban residents in the United States. Medical costs, education, and long-term care are all expected to inflate at a rate higher than the CPI-U in the coming years.

If transferred or purchased inappropriately, otherwise tax-free life insurance benefits may be subject to estate taxation. This demonstrates the need for: (A) tax planning as a part of the comprehensive financial planning process (B) an engagement letter that clearly indicates insurance and tax planning are out-of-scope for financial planners (C) an Irrevocable Life Insurance Trust for any client considering the purchase of life insurance (D) asking clients to sign a waiver of liability for any advice given outside the planner's area of expertise

(A). The complexities of life insurance taxation demonstrate the need for tax planning as a part of the comprehensive financial planning process. Such planning should be in-scope for any comprehensive engagement. And while it is true that an ILIT would help avoid estate taxation, (C) is incorrect because relatively few clients purchasing life insurance should consider an ILIT. (D) is incorrect because it violates the ethical standard of competence. If a planner is not able to give advice on a subject, she should seek outside assistance rather than giving advice outside of her areas of expertise.

Which of the following items would be included in a decedent's gross estate for federal estate tax purposes? (A) a life insurance policy on the decedent's life that was transferred by the decedent 2 years earlier to an irrevocable trust for the benefit of her children (B) the right given by the homeowner several years ago to a decedent to use the homeowner's beach house for the decedent's life (C) real estate given to the decedent by an aunt that, in accordance with the aunt's will, passes to the decedent's sister at the decedent's death (D) property in a trust established by the decedent's father for his grandchildren with the decedent and the decedent's sister as co-trustees of the trust

(A). The life insurance policy is included in the gross estate because the transfer was less than 3 years prior to the decedent's death (Sec. 2035). (B) is incorrect because the decedent merely had a life estate in the beach house which ended when the decedent's life ended, leaving no value to pass to survivors. (C) is incorrect because it is a life estate that the decedent could not transfer at death. (D) is incorrect because the decedent had no beneficial interest in the trust.

Mark and Britney have resided in a community-property state for their entire marriage. Prior to their marriage, Britney had a rollover IRA from a previous employer valued at $50,000. Mark inherited a personal residence prior to their marriage, and they continued to live in this residence after they were married.The residence is currently valued at $400,000. Subsequent to the marriage,they accumulated a brokerage account totaling $300,000. What is Britney's share of the community property? (A) $150,000 (B) $200,000 (C) $350,000 (D) $400,000

(A). The property acquired before marriage is not community property. The only community property in this case is the brokerage account acquired after being married in the community-property state. Britney's share of this is one-half of the $300,000.

What are the three types of structured communication? (A) interviewing, counseling, advising (B) counseling, directive interview, advising (C) advising, counseling, planning (D) interviewing, directive interview, nondirective interview

(A). The three primary forms of structured communication are interviewing, counseling, advising. Directive and nondirective interviews are parts of the interviewing communication structure.

A special needs minor child has been made the beneficiary of a uniform gifts to minors account (UGMA) as a result of a gift from grandparents. Which of the following statements concerning strategies that should be employed to reduce the child's resources for the purposes of the SSI or Medicaid qualification tests is correct? (A) Have a guardian for the minor disclaim the gift to the UGMA. (B) Have the custodian of the account expend funds for the benefit of the minor beneficiary to reduce the size of the account. (C) Withdraw all funds from the account and return the funds to the grandparent who made the gift. (D) Retitle the account in the name of a sibling of the special needs individual.

(B). (A) is incorrect because state law would probably make it impermissible to disclaim resources available to the minor. In any event, this may not be something a guardian should do for the best interests of the beneficiary of an account. (C) is incorrect because it would be a fraudulent transfer to take the funds away from the beneficiary of the account. (D) is incorrect for the same reason as the previous explanation. In effect, retitling the account is stealing from the beneficiary of the account.

When a stock's share price moves more than the market (A) Beta = 0 (B) Beta > 1 (C) Beta < 1 (D) Beta = 1

(B). (A) is incorrect because when Beta=0, the volatility is consistent with a risk-free asset. (C) is incorrect because when Beta<1, the stock's price moves opposite the market. (D) is incorrect because when Beta=1, the stock's price moves in line with the market.

All of the following statements concerning a Sec. 6166 installment payment of estate tax are correct EXCEPT: (A) A closely held business in excess of 35 percent of the adjusted gross estate must be included in the gross estate to qualify. (B) The first 10 installment payments are interest only. (C) The estate must be kept open during the installment period. (D) The estate may use Sec. 303 redemptions to make the installment payments.

(B). (B) is incorrect because the first 5 years of payments are not interest only.

All of the following statements concerning the advantages of the irrevocable life insurance trust (ILIT) are correct EXCEPT: (A) The grantor can use the federal gift tax annual exclusion to shelter some or all of the gifts to the trust for premium payments from taxable gifts. (B) The grantor can retain the right to access the cash surrender value for emergency needs. (C) The proceeds will escape taxation in the grantor's gross estate for federal estate tax purposes. (D) The trustee can use the insurance proceeds to increase the liquidity position of the grantor's gross estate.

(B). (B) is incorrect because the right to access cash surrender value is a valuable incident of ownership and places the proceeds of the insurance in the grantor's gross estate, which is exactly what the ILIT is designed to avoid.

All of the following entity types are required to file documents with the state when the entity is formed EXCEPT: (A) corporation (B) general partnership (C) LLC (D) limited partnership

(B). A general partnership is not required to file documents with the state at the time of formation. Although it is not required, we would certainly recommend a formal partnership agreement be executed between the partners.

Ignoring the annual per-donee exclusion, all of the following transfers are taxable gifts for federal estate tax purposes EXCEPT: (A) A grandfather reimburses his grandson $88,000 for his college tuition costs. (B) A neighbor gratuitously performs $16,000 worth of valuable construction, electrical, and plumbing services for the benefit of his friend. (C) A mother lends her daughter $150,000 interest-free for a period of 4 years. (D) A creditor cancels the $60,000 promissory note of a relative who recently lost her job.

(B). A reimbursement of college expenses to the student, a substantial interest-free loan, and the cancellation of a sizable promissory note by a relative are all taxable gifts. Services, however, do not constitute taxable gifts for federal gift tax purposes.

A disabled minor child lives with his parents. All of the following statements concerning the child's eligibility for Supplemental Security income (SSI) and Medicaid are correct EXCEPT: (A) The income and resources of the parents might be deemed available to the minor child for the purposes of the eligibility tests. (B) The child's resources are ignored for the purposes of the eligibility tests until the child reaches the age of majority. (C) Gifts or inheritances left to the child should be carefully planned. (D) Medicaid is very important under these circumstances even if the child is covered under the parents' health insurance as a dependent.

(B). All resources available to the SSI and Medicaid applicant will affect the eligibility tests regardless of the age of the applicant.

All of the following are true regarding ABLE accounts and special needs pooled trusts EXCEPT: (A) An ABLE account provides the account holder more flexibility than a special needs pooled trust. (B) An individual who has a special needs trust may not also maintain an ABLE account. (C) An ABLE account is less expensive to maintain than a special needs pooled trust. (D) An individual wanting more control over the distribution of funds will likely choose a special needs pooled trust rather than an ABLE account.

(B). An individual's eligibility for an ABLE account is not altered by whether or not that individual is a beneficiary of a special needs trust. (A) is correct because an ABLE account is not confined by the terms set forth in the trust document. The manager of the account has more flexibility to disperse funds in the account than a trustee. (C) is correct because an ABLE account is not subject to trustee fees. (D) is correct because an individual who wishes to maintain control can ensure his or her wishes are carried out through a trust document, but will not be able to maintain control in an ABLE account.

A drawback of case studies is that they don't prepare students for: (A) Analyzing client data (B) Adjusting a plan when clients' goals change (C) Prioritizing recommendations (D) Integrating multiple financial planning topics

(B). Case study clients are static; unlike real people, their goals, needs, and life situations never change. Case studies are still useful, however, in that they prepare students for analyzing data, prioritizing recommendations, and integrating topics into a single, cohesive plan.

Which of the following risks is most appropriately managed with insurance? (A) A low-frequency, low-severity risk (B) A low-frequency, high-severity risk (C) A high-frequency, low-severity risk (D) A high-frequency, high-severity risk

(B). Catastrophic, low-probability events should be insured against. The fundamental purpose of insurance is to shift the risks of loss from the client to the insurance company. While clients would hope to transfer the risk of high-frequency, high-severity risks as well, insurance companies are unwilling to take on these risks themselves; if they were, premiums would be prohibitively expensive anyway.

Which of the following terms best describes the process of comprehensive financial planning? (A) product-driven (B) client-oriented (C) single purpose (D) multi-purpose

(B). Comprehensive financial planning is a client-oriented process that focuses closely on a client's goal before attempting to discover solutions.

An advisor who hopes to involve her clients in the process of developing recommendations might focus more on which of the following approaches to communication? (A) Advising (B) Counseling (C) Directive interviewing (D) Nondirective interviewing

(B). Counseling can help clients to generate "self-advice," which can inform recommendations. (A) is incorrect because advising is less likely to involve input from clients. (C) and (D) are incorrect because interviews are more appropriate for generating data than for developing recommendations.

Dee Rigor, a widow, made the following gifts in 2018: $60,000 in cash to her nephew; $25,000 in cash to her favorite qualified animal rescue charity; stock worth $30,000 to her grandson (originally purchased for $5,000); bonds worth $190,000 to The American College (originally purchased for $90,000). What is the total amount of taxable gifts made by Dee? (A) $50,000 (B) $60,000 (C) $277,000 (D) $335,000

(B). Each gift will qualify for the annual exclusion and the remainder of the gifts to the animal charity and The American College will qualify for the charitable gift tax deduction. Therefore only $45,000 of the gift to her son and $15,000 of the gift to her grandson will be taxable gifts.

All of the following are key elements of risk management EXCEPT: (A) life insurance (B) estate planning (C) health insurance (D) property and casualty insurance

(B). Estate planning is not considered an element of risk management, while forms of insurance - such as answers (A), (C) and (D) are.

Which of the following is a gift for federal gift tax purposes? (A) A painting by a world-renowned artist is delivered to the owner's nephew with the understanding that the painting can be taken back by the aunt if she wishes at a future time. (B) A father forgives the remaining balance of a significant loan made to his son 8 years ago. (C) A landscaper provided extensive landscaping for a good friend's one-acre property without receiving any compensation. (D) An uncle promises his niece an around-the-world trip when the niece graduates from college.

(B). Forgiveness of money owed is considered a gift unless it is in a business context. (A) is incorrect because, since the aunt made a revocable gift, it is not a completed gift and therefore is not a taxable gift. (C) is incorrect because gratuitous services are not taxable for gift tax purposes. (D) is incorrect because a mere promise does not constitute a taxable gift.

Which of the following statements is correct regarding confirmation bias? I. Confirmation bias does not affect highly intelligent investors. II. Confirmation bias affects the depth and breadth of information people seek when they are forming an opinion or making a decision. (A) I only (B) II only (C) Both I and II (D) Neither I nor II

(B). I is incorrect because confirmation bias affects even the most intelligent individuals. II is correct because of the narrowing effect of confirmation bias on fact finding and decision making.

The following are common behaviors that violate modern portfolio theory (MPT) EXCEPT: (A) Not all investors hold only the market portfolio. (B) Increasing public information makes market activity more efficent, benefitting investors. (C) Some investors invest in active strategies that rely on market inefficiencies to provide excess returns. (D) Some investors hold portfolios with very little diversification and purposely concentrate holdings.

(B). Increased public information assists investors in creating more efficient portfolios by helping to value stocks more efficiently, according to MPT.

Which of the following statements regarding community-property issues for divorce purposes are correct? I. Community property has automatic right of survivorship. II. Paying divorce costs from separate property can convert at least part of that property into community property. (A) I only (B) II only (C) Both I and II (D) Neither I nor II

(B). Joint tenancy has automatic right of survivorship under operation of law. Community property can be willed. Divorcing spouses must be careful to distinctly account for community property and separate property. Using separate property to pay for community-property expenses can convert at least part of that separate property into community property.

Leigh, a financial planner, has just presented her recommendations to her client, Walter. Walter isn't happy: Leigh says he should save more, retire later, and take less retirement income than he expected. He claims that her assumptions are too pessimistic. What should Leigh do? (A) Adjust her inflation and investment return assumptions to appease Walter. (B) Explain how she chose her assumptions and the risks associated with using assumptions that are too aggressive. (C) Keep her assumptions the same, but adjust her recommendations to be less pessimistic. (D) Terminate her professional relationship with Walter.

(B). Leigh's assumptions should be reasonable and based on historical data. Adjusting assumptions or recommendations to appease a client runs the risk of that client's plan failing. If Walter and Leigh cannot agree on an exact number, Walter should know that it is preferable to make assumptions that are somewhat too conservative than too aggressive.

Melvin is an independent, comprehensive financial advisor who focuses on the needs of small business owners. At the start of the new year and with tax filing season approaching, what might Melvin do to provide the best service for his clients? (A) Take night classes that will culminate in his earning a CPA designation. (B) Establish a working relationship with a CPA who also serves small business owners. (C) Require all of his clients to retain the services of a CPA or else sever the engagement. (D) Clearly communicate that he does not provide tax planning as a part of his services.

(B). Melvin's clients would benefit from his working in a team with a CPA. (A) is incorrect because Melvin does not have time to become a tax expert. (C) is incorrect because some clients may not need a CPA; moreover, it's not in the clients' best interest to terminate the financial planning engagement right before tax season. Finally, (D) is incorrect because tax planning should be a part of any comprehensive financial planning engagement.

Which of the following situations concerning the nonrecognition of gain rules provided by IRC Sec. 1041 dealing for property settlements pursuant to a divorce is correct? (A) The nonrecognition rules apply to a transfer to a former spouse who is a nonresident alien. (B) The nonrecognition rules apply to a sale to a former spouse. (C) The nonrecognition rules apply to transfers from the time of divorce until a decade later. (D) The nonrecognition rules apply to a redemption of the former spouse's stock in a family corporation.

(B). Per IRC Sec. 1041, property settlements represent a division of existing property and are not taxable.

A divorcing couple, Olivia and Philipe, is trying to determine how much child support will be due. The monthly cost to support their child is considered to be $4,000. Olivia will maintain custody of the child and has net earnings of $150,000 per year. If Philipe has net earnings of $50,000 per year, how much child support will he pay each month? (A) $0 (B) $1,000 (C) $2,000 (D) $3,000

(B). Philipe will pay Olivia $1,000 a month. The first step in this process is to add their net earnings together ($150,000 + $50,000 = $200,000). The next step is to determine the percent that the noncustodial spouse (Philipe) contributes to that total. He contributes 25% ($50,000 / $200,000) and must therefore contribute 25% of the total cost of raising the child. That cost is $4,000 a month, so Philipe must contribute $1,000 (25% of $4,000).

Each of the following statements is a correct description of retirement income planning EXCEPT: (A) Retirement income planning is a distinct field from "retirement planning" and requires different subject matter expertise than what may be needed to be a retirement planner. (B) It is a relatively old field within the financial services industry that focuses on assisting a client to accumulate assets for use during retirement. (C) Retirement income planning identifies and creates strategies to eliminate or minimize risks that could disrupt the retirement income plan. (D) A retirement income planner must devise a plan to address a shortfall if a client has insufficient resources to provide the required post-retirement income.

(B). Retirement income planning is a relatively new field within the financial services industry and requires a broad range of subject matter expertise.

Which of the following are examples of satisficing? I. Leah has invested on her own for over 20 years. She is comfortable researching her investments and has an above average success rate finding solid large-cap value stocks. She just chose to invest in stock A (from options: A, B, and C). She is confident in the leadership team (they have a track record of fiscally responsible decisions) and the core ratios are all within her comfort zone. II. Craig, 35, is confused about his 401k investment options. He was recently heard asking, "What is a stock?" He asked a friend what that friend would choose if the friend were Craig. The friend suggests a mutual fund managed for people Craig's age. It is more aggressive now, and gets more conservative as Craig gets older. Craig doesn't really understand what his friend is talking about, but trusts the friend, so that is the fund he chooses. (A) I only (B) II only (C) Both I and II (D) Neither I nor II

(B). Satisficing is making the "good enough" decision. If the decision is based on a well-reasoned process, it is not satisficing. I is not an example of satisficing because Leah employs a practiced, well-reasoned decision-making process. Statement II is satisficing because Craig's decision may not be the optimal choice, but it is the best option available (the "good enough" decision) given the information, time, and Craig's decision-making capacity.

All of the following statements concerning gifts to minors are correct EXCEPT: (A) The gift generally involves some complexity since state laws often restrict the titling of property in the minor's name. (B) The annual gift tax exclusion is unavailable unless the minor receives the property outright. (C) Gifts to a Uniform Transfers to Minors Act (UTMA) custodial account provide for the outright distribution to the minor when the minor reaches the age of majority. (D) Gifts to a Sec. 2503(c) minors trust permit the trustee to accumulate and reinvest income.

(B). Several types of transfer mechanisms are available for gifts to minors that both qualify for the annual gift tax exclusion and restrict the minor's current access to the funds.

Investors seeking return in excess of what the market would bear (A) would receive compensation for the risks that could have been avoided through diversification by adding unsystematic risk, and would be compensated for the degree of beta they were willing to accept (B) would not receive compensation for the risks that could have been avoided through diversification by adding unsystematic risk, but would be compensated for the degree of beta they were willing to accept (C) would not receive compensation for the risks that could have been avoided through diversification by adding unsystematic risk, and would not be compensated for the degree of beta they were willing to accept (D) would receive compensation for the risks that could have been avoided through diversification by adding unsystematic risk but would not be compensated for the degree of beta they were willing to accept

(B). Since diversification is possible through the addition of unsystematic risk, an investor would receive no additional compensation for taking the risk of an undiversified portfolio. However, the investor should be compensated for his or her willingness to assume risk related to Beta. (A) is incorrect because it states that the investor would receive compensation for the risks. (C) is incorrect because it states that the investor would not be compensated for the degree of Beta. (D) is incorrect because it states that the investor would receive compensation for the risks and would not be compensated for the degree of Beta.

Which of the following are appropriate mechanisms for transferring assets if a donor, who is young and in good health, wants to minimize his taxable estate while still receiving income from the assets for some time? I. Creating an irrevocable trust that allows his friends' children to receive income for the next 10 years, after which the corpus of the trust will be distributed to a qualified charity II. Creating a trust that allows him to receive income for 10 years, and at his death, the trust assets pass to a qualified charity (A) I only (B) II only (C) Both I and II (D) Neither I nor II

(B). Since the donor is in good health, he will likely survive the trust term, and the trust assets will not be included in his estate. Statement I is not an appropriate choice because although he accomplishes minimizing his taxable estate, he cannot receive income from the trust property.

A trustee pays all income and principal as necessary for the beneficiary's support. Which of the following statements concerning the impact of the trust on potential qualification of the beneficiary for SSI and Medicaid is (are)correct? I. The trust is exempt from the income and resource tests of SSI and Medicaid because it was created and funded by a third-party. II. The trust terms should be examined to determine if it is possible to distribute the trust funds to a newly created special needs trust. (A) I only (B) II only (C) Both I and II (D) Neither I nor II

(B). Statement I is incorrect because a trust that must distribute for the support of an applicant for SSI or Medicaid is counted as a countable resource to the extent such funds are available for support.

Which of the following statement(s) is (are) true regarding the "unified" system of estate and gift tax? I. The same federal exemption amount is shared for gifts, estates, and GST taxes. II. The applicable credit amount applies to lifetime and deathtime transfers. (A) I only (B) II only (C) Both I and II (D) Neither I nor II

(B). Statement I is incorrect because although gift, estate, and GST taxes are all apart of the unified transfer system, transfers subject to GST tax use a separate federal exemption amount.

Which of the following correctly describes the tax treatment of property settlement pursuant to a divorce? I. It is a nontaxable event because the transfers are treated as gifts between married spouses. II. It is a taxable event if not performed within the designated time frame. (A) I only (B) II only (C) Both I and II (D) Neither I nor II

(B). Statement I is incorrect because per IRC Sec. 1041, property settlements pursuant to a divorce represent a division of existing property and are not taxable.

Which of the following statements concerning a cross-purchase buy-sell agreement effective at the death of an owner of an LLC is (are) correct? I. The LLC should own a policy covering the life of each owner participating in the agreement. II. Each owner participating in the agreement binds his or her estate to sell the membership interest at death to the surviving owners. (A) I only (B) II only (C) Both I and II (D) Neither I nor II

(B). Statement I is incorrect because the individual owners are the purchasers and each owner should own a policy on the life of any owner he or she is obligated to buy out.

A couple is contemplating divorce. Which of the following statements concerning their predivorce responsibilities to each other is (are) correct? I. Each spouse should protect his or her position by immediately withdrawing funds from the joint accounts. II. The joint assets are subject to division and should be protected by both spouses. (A) I only (B) II only (C) Both I and II (D) Neither I nor II

(B). Statement I is incorrect because the only requirement is that each spouse preserves the joint accounts and will be required to account for any other actions during the process of divorce.

A wife makes outright gifts of $55,000 to her son in 2019, and her husband agrees to split the gifts with her. Which of the following correctly states the amount of the taxable gifts? (A) wife - $20,000, husband - $20,000 (B) wife - $12,500, husband - $12,500 (C) wife - $27,500, husband - $27,500 (D) wife - $55,000, husband - $0

(B). The calculation of taxable gifts is as follows: wife/husband, total gifts: $27,500/$27,500. Less 2019 annual exclusion $15,000/$15,000. Total taxable gifts: $12,500/$12,500.

All of the following statements concerning the child and dependent care credit are correct EXCEPT: (A) The amount of the credit is dependent on AGI and the number of qualifying individuals. (B) The maximum amount of qualifying expenditures cannot exceed $5,000 for one qualifying individual and $10,000 for two or more qualifying individuals. (C) The credit is reduced for upper-income taxpayers. (D) The credit is available with respect to care for a dependent under the age of 13 or an incapacitated dependent of any age (including a spouse or special needs child).

(B). The dollar limit is $3,000 for one qualifying individual and $6,000 for two or more qualifying individuals.

The first assumption of neoclassical economics is what? (A) One cannot maximize utility. (B) We have the full internal knowledge to make choices that reflect our preferences. (C) Market participants can outperform the market based on their preferences. (D) Model behavior and actual behavior of investors mirror each other in market settings.

(B). The first assumption of neoclassical economics is that we have all the information we need to make choices that reflect our preferences. Full information includes full internal knowledge and a knowledge of our preferences.

The floor for medical expense deductions is set at what percentage of the taxpayer's adjusted gross income (AGI) for 2019? (A) 1 percent (B) 10 percent (C) 7.5 percent (D) 12 percent

(B). The floor applicable to medical expense deductions is 10 percent of AGI in 2019 per the TCJA Tax Reform Act.

All of the following statements describe key issues in Step 3 of the retirement income planning process EXCEPT: (A) deciding whether or not to use the ratio-replacement method or the expense method to calculate projected retirement expenses (B) explaining the Monte Carlo method to the client (C) finding and using a software application that is sufficiently comprehensive to integrate all of the possible changes in expenses that a client could encounter during retirement (D) securing the client's cooperation in preparing a list of the expected expenses for the first year of retirement expenses

(B). This is not a consideration in Step 3 of the retirement income planning process.

An investor has the choice between a 90% chance of winning $3,000 or a certain payment of $500. If the investor chooses the $500 certainty, whatbehavior does this illustrate in behavioral finance? (A) Uncertainty (B) High risk aversion (C) Decision anxiety (D) Inertia

(B). This is perhaps the most important bias in behavioral finance-- the tendency of people to react more strongly to losses than to gains. In other words, minimizing risk is much more important than increasing gain.

All of the following statements correctly describe an ABLE account EXCEPT: (A) ABLE account funds may only be used for "qualified disability expenses," such as housing, education, and health care expenses related to the individual's disability. (B) Contributions to an ABLE account are limited to $15,000 per year, per donor. (C) Contributions made to the ABLE account are not tax-deductible. (D) Investments in the ABLE account can be adjusted up to two times per year.

(B). Total annual contributions are limited to $15,000 per year.

The semi-strong form of the efficient market hypothesis asserts all of the following EXCEPT that: (A) technical analysis of past price data will not be fruitful because prices reflect historical information (B) investors cannot generate abnormal returns using information releases because prices reflect any public information in addition to past price data (C) company insiders cannot devise a strategy to consistently generate abnormal returns, because prices reflect private information as well (D) earnings statements and media reports are not sufficiently suitable for providing abnormal returns

(C), which is the "strong form." (A) reflects the philosophy of the "weak form," therefore is included in the "semi-strong form." (B) and (D) reflect the philosophy of the "semi-strong form."

Tom and Mary were granted a divorce on 10/5/17. As a result, Tom moved to another city to take a job and start over. He sold his house for $800,000on May 1, 2018, just 6 months after he purchased it for $500,000. When filing his 2018 tax return, how much of the gain must Tom include on his 2018 tax return? (A) $0 (B) $62,500 (C) $237,500 (D) $300,000

(C). $800,000-$500,000 = $300,000 profit. The 2-year rule applies, and $250,000 is the single exclusion on the sale of a home. $250,000 x 6mo/24 mo = $62,500 profit excluded. Balance of profit included for income tax = $237,500. Note that the $250,000 is the single exclusion on the sale of a home.

Which of the following statements concerning the federal tax treatment and compensation of the owners of a partnership is correct? (A) A partnership must distribute all of its taxable income to the partners by the close of the tax year, because it is a pass-through entity for tax purposes. (B) Partners are treated as employees of the partnership for federal income tax purposes if they provide substantial services. (C) Service partners are often compensated through guaranteed payments. (D) A partnership should be formed if the owners are to participate in complex nonqualified fringe-benefit packages.

(C). (A) is incorrect because distributions made to partners are a discretionary decision made by the managing partners. The pass-through nature of taxation merely provides that the partners are taxed on the partnership's taxable income regardless of whether or not they received distributions of cash or property. (B) is incorrect because the partners are treated as self-employed for federal income tax purposes. (D) is incorrect because complex nonqualified fringe-benefit packages are generally appropriate only in regular corporations.

Which of the following statements concerning the sale of the business interest for a private annuity is correct? (A) The gain on the sale is recognized by the seller over the seller's remaining life expectancy. (B) All income from the sale proceeds is taxed at ordinary income tax rates. (C) No part of the annuity is included in the seller's gross estate because the annuity terminates at the seller's death. (D) A private annuity transaction is often formed between an unrelated buyer and seller.

(C). (A) is incorrect because tax regulations require that the entire gain be taxable at the time of the sale. (B) is incorrect because the gain is taxed at capital gains tax rates, and the interest element is taxed at ordinary income tax rates. (D) is incorrect because almost all private annuity sales are between family members.

Mr. and Mrs. Ripon have a special needs grandchild with severe disabilities. In their wills, they provide for a testamentary trust for the special needs grandchild that will provide the grandchild with all income for life as well as the principal as necessary for the support of the grandchild at the discretion of a corporate trustee. Which of the following statements correctly describes the impact of this trust arrangement? (A) The trust is treated as an available asset of the grandchild for Medicaid and SSI purposes as soon as the wills are executed. (B) The trust is treated as an exempt asset when the grandparents die and the trust is funded with respect to the grandchild for SSI and Medicaid purposes since it was created by a third party. (C) The trust is treated as an available asset of the special needs grandchild with respect to Medicaid and SSI when it is funded at the death of the grandparents. (D) The trust has no impact whatsoever on the grandchild's qualification for SSI or Medicaid benefits.

(C). (A) is incorrect because the trust is not funded until the grandparents' death and cannot be an available asset until funded. (B) and (D) are incorrect because a trust that provides for a mandatory income distribution will provide income that is countable under the SSI and Medicaid rules. In addition, the availability of principal for the support of the grandchild would cause the trust fund to be treated as an available asset.

Which of the following statements concerning the role of professional advisors and the estate planning process for the closely held business owner is correct? (A) Only one advisor is typically used to avoid excessive costs and prevent conflicts between advisors. (B) The advisor should avoid communicating with the business owner's family members or other interested parties to avoid conflicts of interest. (C) A team of advisors tends to work more effectively if there is one who focuses the team and coordinates the completion of the individual tasks. (D) Most professional advisors will be highly skilled in all the tasks associated with business succession and estate planning.

(C). (A) is incorrect because there are numerous skills required and often a team of advisors will be required. (B) is incorrect because the advisor or advisors are often used to help communicate a complex issue to family members and/or key employees. (D) is incorrect because there are many skills required in this process including legal issues, business and personal tax issues, and effective communication, and it is unlikely that every advisor will possess all the required skills.

Which of the following statements concerning the federal income tax treatment of a regular corporation is correct? (A) The accumulated earnings tax might be imposed if the corporation has accumulated earnings and profits below $250,000 if there is no reasonable business need for the accumulation. (B) The corporate alternative minimum tax (AMT) could be imposed on a corporation with low normal income tax liability regardless of the size of a corporation. (C) A corporation may be advantageous from a tax standpoint if small amounts of corporate earnings are to be reinvested in the corporation. (D) A corporation has the same tax tables as individuals because Congress intended to eliminate the advantage of using the corporation as a separate taxpayer.

(C). (A) is incorrect because there is a minimum safe harbor for accumulated earnings of a business corporation of $250,000. (B) is incorrect because, as a result of the TCJA, there is no longer a corporate AMT. (D) is incorrect because the rates and brackets for corporations and individuals are significantly different.

An advisor notices her clients tend to be somewhat anxious during their meetings. Which action might you recommend to help alleviate her clients' concerns? (A) Use technical language and jargon to showcase expertise. (B) Speak at least half of the time, so clients don't have to control the meeting. (C) Maintain an open body posture and speak in a calm, consistent pace. (D) Avoid making too much eye contact, because it this makes some people uncomfortable.

(C). An open body posture and a calm speaking style can help alleviate clients' concerns. Using technical jargon and dominating the conversation might convey expertise, but these do not tend to put people at ease. Also, (D) is incorrect because maintaining—not avoiding—eye contact helps alleviate clients' concerns.

All of the following will be brought back into the donor's gross estate for federal estate tax purposes EXCEPT: (A) the gift taxes paid last year on a gratuitous transfer of real property (B) a gratuitous transfer of real property 2 months ago to a revocable inter vivos trust (C) an outright gratuitous transfer of real property 2 years ago in contemplation of death (D) a gratuitous transfer of real property 4 years ago with a reserved right to use it for life

(C). An outright gift of property to a donee falls under the general rule of Sec. 2035, which exempts gifts in which the donor has relinquished all dominion and control from inclusion in his or her gross estate.

A 55-year-old client has been a participant in her employer's frozen defined-benefit plan and is now offered the option to take an immediate lump-sum payment during a limited period in lieu of a deferred annuity that would commence no earlier than when she attains the normal retirement age of 65.Which factor would NOT suggest that the client should take the lump sum? (A) The client is in ill health. (B) The payment of the benefit is not insured by any governmental agency. (C) The plan is currently overfunded. (D) The client feels that she could invest the lump-sum payment and increase the value of the benefit at retirement.

(C). An overfunded plan would increase the likelihood that future annuity payments would be made in full. (A) is incorrect because the present value of the annuity payments would be less for a client whose life expectancy is lower than average, so the lump sum would economically be a sound choice. The lump sum might also be needed to pay for high near-term health care expenses. (B) is incorrect because the PBGC only provides partial reimbursement for pension recipients who are eligible to receive high income payments. (D) is incorrect because if the client believes they could receive a higher rate of return from investing the lump sum, they may not prefer the annuity.

The difference between System 1 and System 2 in behavioral finance is that I. System 1 is a brain region known to process lower level, automatic, instinctual types of responses II. System 2 processes higher level, more deliberate responses to a stimulus (A) I only (B) II only (C) Both I and II (D) Neither I nor II

(C). Both I and II are correct and describe the two systems of the two systems theory of behavioral finance.

Investors tend to underperform in mutual funds because of I. return chasing, which tends to lead to underperformance II. tending to weight more recent information, also known as recency bias (A) I only (B) II only (C) Both I and II (D) Neither I nor II

(C). Both I and II are correct.

Which of the following statements concerning contributions to ABLE accounts is (are) correct? I. A beneficiary cannot receive SSI payments if the value in an ABLE account exceeds $100,000. II. The maximum contributions amount will be adjusted annually for inflation. (A) I only (B) II only (C) Both I and II (D) Neither I nor II

(C). Both Statement I and Statement II are correct.

Which of the following assets owned by a Medicaid applicant is (are) exempt from the Medicaid resource test? I. A cash value life insurance policy with a face amount of $1,500 II. A specially designed vehicle used to transport the Medicaid applicant (A) I only (B) II only (C) Both I and II (D) Neither I nor II

(C). Both assets are exempt from the Medicaid eligibility test.

Which of the following statements concerning federal income tax rates is (are)correct? I. The amounts of taxable income in the brackets to which various tax rates apply vary according to the taxpayer's filing status. II. An individual taxpayer's effective marginal tax rate may be different than his or her marginal tax rate. (A) I only (B) II only (C) Both I and II (D) Neither I nor II

(C). Both statements are correct.

Which of the following statements is (are) true regarding Roth IRA conversions? I. If Joe has a $100,000 tax-deferred IRA and converts it this year, he will increase his taxable income by $100,000. II. A Roth IRA conversion must be completed by December 31 in the year in which you are trying to do the conversion. (A) I only (B) II only (C) Both I and II (D) Neither I nor II

(C). Both statements are correct. Taxable income results at the time of a conversion for any tax-deferred amounts. However, there is no 10% penalty tax. Statement II is correct because in order to convert an account, say for 2019, you need to do that conversion by December 31, 2019.

Which of the following should a financial services professional consider during a "needs and savings analysis" as part of the retirement income planning process? I. The client's retirement age and life expectancy II. Inflation, projected investment return, and tax rates (A) I only (B) II only (C) Both I and II (D) Neither I nor II

(C). Both statements combined list the major assumptions that are factored into a "needs and savings analysis."

All of the following are retirement planning considerations unique to blended families EXCEPT: (A) changing qualified plan beneficiary designations to the new spouse after marriage (B) including the custodial parent's new spouse's income in the calculation for student financial aid (C) increasing the business valuation to increase the amount of support given to the custodial parent by the parent retaining the business asset (D) changing the name on all assets transferred to the spouse prior to divorce

(C). Business valuations must be calculated by an expert utilizing 3-5 years of detailed tax returns and other records to determine valuations as precisely as possible based on past earnings and often future estimated earnings. In some cases, forensic accounting may be necessary if foul play or fraud is suspected. (A), (B), and (D) are all correct answers.

All of the following statements correctly describe ABLE accounts EXCEPT: (A) An ABLE account may be maintained for the benefit of the special needs individual only. (B) An individual is not required to pass a means or resource test in order to establish an ABLE account. (C) ABLE accounts cannot be funded with private contributions from individuals. (D) Contributions to an ABLE account are not tax-deductible, and income earned on the savings account is tax-exempt.

(C). Contributions to an ABLE account can be made with private funds. (A) is correct because the account can be established for the disabled individual or a family member of the disabled individual. (B) is correct because a person is eligible for an ABLE account if the disability onset was before age 26 and if he or she meets the significant limitations test to receive SSI. (D) is correct because while contributions are not tax-deductible, income earned on the account is tax-exempt.

The core idea of CAPM is that all financial assets can be priced based on their risk, defined as uncertainty of their future payout. CAPM recognizes that two assets can each have a high standard deviation, but when combined, these can have a lower standard deviation. This is known as: (A) weighted-average risk (B) risk assessment (C) diversification (D) portfolio utility maximization

(C). Diversification is one of the core principles of CAPM because it reduces risk and improves the efficiency of portfolios. (A), (B) and (D) are incorrect and not part of CAPM theory.

Which of the following risks can be addressed through the purchase of insurance company products (i.e., insurance and annuities)? I. The long-term care risk II. The longevity risk (A) I only (B) II only (C) Both I and II (D) Neither I nor II

(C). Long-term care insurance is used to address long-term care risk and annuity products can be used to help reduce or transfer the risk of longevity onto the insurance company by providing lifetime income if it is a life annuity.

All of the following statements concerning the tax treatment of transfers pursuant to divorces dated on or after January 1, 2019 are correct EXCEPT: (A) Payment of spousal support will not be deductible for federal income tax purposes for the payer spouse. (B) Payment of spousal support will not be treated as income for federal income tax purposes for the payee spouse. (C) Payment of spousal support is treated as a transfer for less than full and adequate consideration with respect to the federal gift tax laws. (D) Transfers pursuant to a divorce property settlement agreement are exempt from recognition of capital gains under Sec. 1041 of the Internal Revenue Code.

(C). Payment of spousal support is a required obligation of the payer spouse and is not treated as a gift.

All of the following are steps in the financial planning process EXCEPT: (A) establish goals (B) analyze and evaluate (C) prospect (D) develop recommendations

(C). Prospecting is not part of the financial planning process. Before employing the process, it is critical that advisors prospect and gather clients.

Cheryl owns a large commercial building that she currently actively manages.She has significantly depreciated the building for federal income tax purposes and has a substantial built-in capital gain. She would like to sell the building to her daughter to try to freeze the value of the building for estate tax purposes.She is comparing the sale of the building to her daughter by using either an installment sale or a private annuity. Which of the following statements concerning these choices is correct? (A) Both transactions will provide Cheryl with the advantage of deferring the long-term capital gain. (B) Both transactions will result in an equivalent cost basis for the building in the hands of the purchasing daughter. (C) Both transactions will immediately remove the building from Cheryl's gross estate once the sale is complete. (D) Both transactions will have identical effects on Cheryl's gross estate regardless of when death occurs.

(C). Statement (A) is incorrect because the private annuity will cause all gain recognition at the time of the sale. Statement (B) is incorrect because the installment sale provides the daughter with a cost basis equal to the purchase price while the private annuity results in a basis limited to the amount of payments actually made by the daughter. Statement (D) is incorrect because the private annuity is automatically excluded from the gross estate at the time of death, whereas any unpaid balance on the installment note will be included in the gross estate of the seller if death occurs before the note is satisfied.

All of the following statements concerning lifetime gifts of closely held business interests are correct EXCEPT: (A) gifts of a closely held business interest can be designed to be eligible for the annual gift tax exclusion (B) the value of a gift of a noncontrolling interest in a closely held business can be discounted for lack of control (C) lifetime gifts provide the advantage of a lower marginal rate than transfers at death, which are subject to the federal estate tax (D) aggregate lifetime taxable gifts must exceed $10 million (indexed for inflation and doubled by the Tax Cuts and Jobs Act in 2017) before any gift taxes will be paid by the transferor

(C). Statement (C) is incorrect because the federal gift and estate tax system uses a uniform set of rates and all taxable transfers ultimately will be subject to a marginal 40 percent rate.

Which of the following statements concerning federal gift tax is (are) correct? I. If X gives an apartment house to a child but retains the right to any rental income from the building for life, the present value of the retained income stream is subject to gift taxation. II. If Y places $1 million in a trust that provides for the payment of all trust income to Y's son for 10 years with the remaining principal directed to Y's daughter, Y has made a gift of the present value of the income stream to his son and a gift of the present value of the remainder interest to his daughter. (A) I only (B) II only (C) Both I and II (D) Neither I nor II

(C). Statement I is correct because Donor retained no rights that would affect the remainder interest to Kate. Statement II is correct because Donor has the right to control and affect the income interests of Abe and Ben, and it is, therefore, an incomplete gift. However, as income is actually paid out to Abe and Ben, the amount paid constitutes a completed gift of a present interest.

Which of the following statements is (are) correct regarding the use of an engagement letter? I. An engagement letter is intended to inform a client as to the scope, fees, and responsibilities of the parties in regard to retaining a financial services professional to provide retirement income planning services. II. Usually an engagement letter cannot be changed unless the client gives consent. (A) I only (B) II only (C) Both I and II (D) Neither I nor II

(C). Statement I is correct because it describes the purpose of the engagement letter. Statement II is correct because the engagement letter is a contract and may only be changed pursuant to its amendment procedures which would likely require some form of client consent.

Which of the following statements concerning the provider of educational services to the special needs individual is correct? I. It is primarily but not always provided by the public school system. II. It is sometimes delivered by a specialized private school for special needs individuals. (A) I only (B) II only (C) Both I and II (D) Neither I nor II

(C). The law requires states to provide free education to children without disabilities and to provide free education to children with disabilities as well. Most of this education is provided by the public schools. However, if the public school system lacks the appropriate facilities, it must bear the cost of placing the special needs individual in the appropriate specialized private school.

John Jones transferred property to his ex-spouse as part of their divorce agreement. The property was several acres of prime real estate in an area that was experiencing a lot of commercial development. Which statement below concerning property basis for the transfer is correct? (A) There is a 25% step up in basis if the ex-spouse becomes disabled. (B) There is a step up in basis if the property is worth more than John paid for it. (C) There is no step up in basis. (D) The marital basis may be increased if the divorce was contentious.

(C). There is no step up in basis. The marital basis is assumed by the ex-spouse who receives the transfer. The fair market value is not considered, nor are any liabilities considered. A step up in basis will only occur at the death of the ex-spouse.

A prospective client is trying to decide whether or not to retire now or wait a couple of years. She has considerable assets in tax-deferred 401(k) and Traditional IRA accounts and wonders if a Roth conversion is right for her. Additionally, she is recently divorced and financially support a child from that relationship who is now in college. She also has a parent in a long-term care facility and is worried about her own potential long-term care needs. Which financial planning approach is the most appropriate for this prospective client? (A) The single-purpose approach (B) The multi-purpose approach (C) The comprehensive approach (D) This client is not a good fit for any financial planner

(C). This client would benefit from a comprehensive approach that considers her holistic, financial, and life situation. While she is only trying to decide whether to retire now or later, her situation is far too complex to focus on that issue alone at the exclusion of her other goals and needs.

In which of the following scenarios would it be most appropriate for a client to consider using a qualified terminable interest property (QTIP) trust from an estate planning perspective? (A) A client has an estate that is worth approximately $20 million and wishes to avoid all estate taxes upon both her passing and the passing of her spouse. (B) A client wishes for her home to be transferred immediately to her biological son at her passing as opposed to being transferred to her current spouse. (C) A client is concerned about conflicts that may arise between her spouse, biological children, and stepchildren after her passing and believes that her biological children are entitled to a larger share of her remainder property after her spouse passes. (D) A client wishes for all of her assets to pass to her current spouse, and she has no concerns about what her spouse does with these assets after the client's passing.

(C). Using a QTIP trust does not avoid estate taxes upon the passing of the second spouse, it does not provide any additional benefit for transfers that are to be made directly to nonspouse beneficiaries at death, and it may not be appropriate in a case where a client is not concerned with what his or her spouse does with the inherited property or assets. A QTIP can be an appropriate tool, especially for blended families, in cases where there are children from different marriages and the client wishes to control the transfer of property after his or her death and the death of the client's spouse.

Killian is a relatively seasoned large-cap growth investor. He is considering an international small-cap growth investment. He has a feeling that the category is poised for good returns, but he isn't sure how to analyze the individual investment (e.g., ratios, exchange rate impact). So he decides to invest in the fund with the best track record. For Killian, this decision is an example of: (A) decision under accountability (B) decision under adversity (C) rational preference (D) satisficing decision making

(D) because a decision made under uncertainty is defined in Prospect Theory as a decision made quickly to get it done or "good enough," or "satisficing." (A) and (B) are incorrect because they don't exist in the discussion of behavioral finance. (C) is incorrect because it describes a decision made after rational considerations. Satisficing is done to "get the job done" quickly without necessarily rational considerations or thought.

Which of the following forms of power of attorney will not take effect until the special needs individual who granted the power becomes disabled or incompetent? (A) special power of attorney (B) durable power of attorney (C) limited power of attorney (D) springing power of attorney

(D). (A) is a type of power of attorney that is limited in scope and time but will take effect immediately. (B) is a power of attorney that takes effect immediately and expressly survives the disability of the principal. (C) is another form of special power.

A prospective client needs help understanding the tax implications of taking income from a complex set of trusts; failure to plan properly may have income tax, generation-skipping transfer tax, and estate tax implications. If you elect to work with this client as a comprehensive financial planner, you should: (A) explicitly indicate that tax planning is out-of-scope of your engagement (B) investigate the relevant tax laws yourself to make an informed recommendation (C) give recommendations based on your current knowledge of general tax laws (D) find a knowledgeable tax attorney who can help inform your recommendations

(D). (A) is incorrect because tax planning is a fundamental part of comprehensive financial planning. (B) and (C) are incorrect because such actions would violate the ethical principle of competency. For such a complex case, it is important to find an expert to help develop informed recommendations.

Which of the following statements concerning the income limitation for payment of benefits under the Supplemental Security Income (SSI) system is correct? (A) 100 percent of earned income is excluded for purposes of the income limitation. (B) 100 percent of unearned income is excluded for purposes of the limitation. (C) The income of a minor child's parents is excluded for purposes of the limitation. (D) The dollar amount of the monthly income limitation is generally equal to the maximum monthly federal benefit payable to SSI recipients and subject to adjustments.

(D). (A) is incorrect because the exclusion for earned income basically applies to the first $65 of earned income for the month, and then to one-half of earned income over $65 for the month. (B) is incorrect because unearned income is generally counted under the income limitation. (C) is incorrect because under the deeming rules, a portion of the income of the parents of a disabled child under age 18 will be counted under the income limitation if the child lives with the parents.

Each of the following are examples/descriptions of the loss-aversion heuristicEXCEPT: (A) a negative emotional response when gaining less than anticipated (B) very small losses are irrationally painful (C) the gambler makes smaller bets, hoping for a few wins (D) more information, although irrelevant, has the same effect on decision assumptions as more informative information

(D). (A), (B), (C) are examples of the loss-aversion bias in behavioral finance.

All of the following statements concerning a qualified domestic relations order (QDRO) are correct EXCEPT: (A) A QDRO cannot require a plan to pay any benefit or option not otherwise provided for in the plan. (B) A QDRO cannot specify which spouse is responsible for payment of taxes. (C) A common mistake of a QDRO is limiting the share of benefits to vested portions only. (D) A QDRO is a nonbinding legal document.

(D). A QDRO is a binding legal document that must be drafted by an attorney.

All of the following statements identify a valid retirement income planning practice consideration EXCEPT: (A) Although the process involves a specific approach to retirement income planning, that process is intended to integrate with the six (6) steps of financial planning as adopted by the CFP Board. (B) Retirement income planning software applications could have limitations that a financial services professional should consider in the development of a retirement income plan. (C) A financial services professional must keep in mind that he is charged with formulating a realistic retirement income plan for the client at hand. (D) A financial services professional should not present alternatives to avoid confusing a client.

(D). A professional should present all of the potential solutions to help a client make the best decisions possible for his or her situation. Only offering one solution would remove the decision making from the client and place it solely on the agent.

As a result of successful business investments made during their 35 years of marriage, Sara and Stuart have acquired substantial wealth. To reduce their estates and minimize taxes, their financial planner suggests they start a gifting program to their children and grandchildren. Assuming they have made no prior gifts, if they elect to gift split, what is the maximum amount Sara and Stuart could give directly to their two grandchildren in 2019, when the GSTT exemption amount is $11.4 million, without incurring any liability for generation-skipping transfers? (A) $19,195,000 (B) $14,708,000 (C) $22,800,000 (D) $22,860,000

(D). Each individual has a lifetime $11.4 million (2019) generation-skipping transfer tax exemption. By gift splitting, a married couple can transfer both of their GSTT exemptions for a total of $22,800,000 (2019) plus annual exclusion amounts of $15,000 (2019); ($30,000 for a married couple) to each donee in a calendar year that is GSTT-free. In this case, $30,000 to each grandchild, for a total of $60,000 to be added to the lifetime GSTT tax exemption, for a total of $22,860,000.

Which of the following statements correctly identifies one of the three primary areas that an engagement letter should address? I. An engagement letter should contain the draft retirement income plan that the engaged financial services professional proposes for the client. II. An engagement letter should address the financial services professional'stheories of investment management. (A) I only (B) II only (C) Both I and II (D) Neither I nor II

(D). I is incorrect because the engagement letter is distinct from the retirement income plan. II is incorrect because the theories of investment management are not a topic for an engagement letter.

Which statement may represent a good strategy during a taxable year in which the client is in a very low federal income tax bracket? I. Contribute to a tax-deferred account. II. A client should consider converting a tax-exempt account into a tax-deferred account. (A) I only (B) II only (C) Both I and II (D) Neither I nor II

(D). I is incorrect because when a client is in a low federal income tax bracket, contributing to a tax-exempt account on an after-tax basis could be advisable and would avoid such amounts being taxed in later years at a higher rate. II is incorrect because converting to a tax-deferred account is not a planning strategy because by definition, a tax-exempt account is the best of both in any case.

If a taxpayer (single tax filing status) had: - 1 qualifying child (age 9) - AGI of $8,000 - Earned income of $4,000 - Qualifying expenses of $2,500 What is the maximum amount of qualified employment-related expenses that would be used to compute the child and dependent care credit? (A) $8,000 (B) $4,000 (C) $3,000 (D) $2,500

(D). In determining the amount of qualified employment related expenses needed to compute the child and dependent care credit, remember that we are limited to the lesser of earned income of the lower earning spouse ($8,000), actual expenses incurred ($2,500), or $3,000 for one child/$6,000 for two or more children. In this case, the actual expenses incurred were the lowest and therefore, the maximum amount of eligible expenses.

The use of an ILIT in estate planning can offer all of the following advantages in estate planning EXCEPT: (A) The ILIT can make a lifetime gift that may be sheltered completely by the grantor's annual gift tax exclusions. (B) The ILIT is a safe way to avoid estate taxes with a uniquely appreciating asset. (C) Probate expenses and publicity are avoided. (D) The grantor can change beneficiaries at will.

(D). In order for the insurance policy contained within the ILIT trust to be excluded from the grantor's estate, the grantor must have no affiliation with the life insurance policy and can not change beneficiaries or pay premiums. The grantor can make gifts of cash to the ILIT trust to pay the premium on the policy. (A), (B), and (C) are correct and explain some of the advantages of using an ILIT in estate planning for sheltering estate tax and gift tax.

Ripley creates a trust for her niece, Newt. Which of the following transfers will incur the GSTT? I. A direct annual gift to Newt of $45,000 II. A taxable termination to Newt of the trust corpus when the trust terminates in 10 years (A) I only (B) II only (C) Both I and II (D) Neither I nor II

(D). Neither are taxable transfers under the GSTT. Lisa is not two generations below Ripley, so the transfers are not eligible for GSTT.

Advantages of an irrevocable life insurance trust (ILIT) include which of the following? I. Income taxes are shifted to the trust's beneficiaries prior to the grantor's death. II. Gift taxes are avoided regardless of the amount of the annual premium contributed to the ILIT. (A) I only (B) II only (C) Both I and II (D) Neither I nor II

(D). Neither statement is correct. I is incorrect because there will be no income taxes until the insurance proceeds are received and the trust fund is invested in income producing investments. II is incorrect because gift taxes can only be avoided if the gifts are below the exclusions and exemption amounts.

All of the following are spousal support requirements EXCEPT: (A) Payments can be made either in a lump sum or in an installment (B) Payments cannot be considered child support. (C) ) There is no set of established rules to determine spousal support requirements based on, for example, length of marriage or income level. (D) If payments end at the death of the payee spouse, the balance of those payments can be made to payee's estate.

(D). Payments must end at the death of the payee spouse.

Two equal shareholders of a closely held corporation have formed a cross-purchase buy-sell agreement funded with life insurance. Which of the following statements concerning this transaction is correct? (A) The buy-sell agreement should be funded with life insurance owned by the corporation. (B) The buy-sell agreement has a business purpose and the premiums for the life insurance funding the agreement are a deductible expense. (C) The insured's estate should be the designated beneficiary of the policy proceeds. (D) The survivor will have an increased cost basis in his or her newly acquired stock equal to the purchase price paid for the stock.

(D). Statement (A) is incorrect because a cross-purchase buy-sell agreement should have each shareholder owning insurance on the life of the other shareholder who must be bought out. Statement (B) is incorrect because the premiums are not deductible because the proceeds will be received income tax free. Statement (C) is incorrect because the shareholder who must purchase the other shareholder's interest should be the designated beneficiary of the proceeds.

Two individuals are planning to form a company to produce software applications. They anticipate the need for significant capital and will have to compensate passive owners. Which of the following entities is most likely to be appropriate for this venture? (A) A business trust (B) A general partnership (C) An S corporation (D) A limited partnership

(D). Statement (A) is incorrect because this was not one of the entity choices discussed in this course and has no track record. Statement (B) is incorrect because it is presumed that all general partners have management authority and will be active. Statement (C) is incorrect because an S corporation can only have one class of ownership and would be inappropriate for passive investors.

Which of the following statements is (are) correct concerning the Medicare program? I. There is typically no additional cost to enroll in a plan under Part C of Medicare. II. Medicare participants must be enrolled in a Medicare Advantage Plan in order to obtain prescription drug coverage under Medicare. (A) I only (B) II only (C) Both I and II (D) Neither I nor II

(D). Statement I is incorrect because coverage under Part C usually requires a monthly premium. II is incorrect because prescription drug coverage is available to all Medicare participants.

Which of the following statements concerning the transition of a closely held business owner to retirement is (are) correct? I. It is important to maximize the current owner's compensation versus the compensation of key employees. II. A successor should not be identified until as close to retirement as possible to prevent attrition among current employees. (A) I only (B) II only (C) Both I and II (D) Neither I nor II

(D). Statement I is incorrect because it is important to balance the future needs of the business, including the motivation of successors and key employees. Statement II is incorrect because it is better to start early to identify and train the successor.

In which of the following situations is (are) the disclaimer(s) effective? I. The beneficiary of the property notifies the executor of his or her intention to disclaim the property in writing, in a timely manner, and specifically notes that the property should not pass to her children. II. The executor is a cousin of the beneficiary. Upon hearing from his aunt that the beneficiary wished to disclaim, the executor promptly gave the property to a qualified charity. (A) I only (B) II only (C) Both I and II (D) Neither I nor II

(D). Statement I is incorrect because the beneficiary cannot influence who the property should or should not pass to. Statement II is incorrect because the executor must receive the disclaimer in writing.

Which of the following entity types would cause all owners of the business to have unlimited liability for damages caused by the business activity? I. S corporation II. LLC (A) I only (B) II only (C) Both I and II (D) Neither I nor II

(D). Statements I and II are incorrect because both entity types provide limited liability for all owners of the entity. Only a sole proprietorship and general partnership do not shield all owners from liability for business activities. In the case of a limited partnership, only the general partner(s) have unlimited liability.

Renée and Sebastian are entering into a divorce. They each incur $10,000 of legal fees for their attorneys. Renée pays the costs for both attorneys. For the purposes of property settlement, they incur $8,000 of valuation appraisal fees. Again, Renée pays this entire cost. Which of the following statements concerning this transaction is (are) correct? I. Renée can deduct the $10,000 of legal fees incurred on Sebastian's behalf from her federal income taxes. II. Renée can deduct the valuation appraisal fee of $8,000 from her federal income taxes. (A) I only (B) II only (C) Both I and II (D) Neither I nor II

(D). Statements I and II are incorrect because these fees associated with the cost of divorce are not deductible for federal income tax purposes.

An advisor who has a more expressive social style has started working with a client who has a more analytical social style. Given their differences, which of the following might the advisor do to make the client more comfortable? (A) Hire an associate with an analytical style and bring that person to every client meeting. (B) Terminate the professional relationship with the client. (C) Schedule fewer meetings and phone calls with the client. (D) Ask the client what would make them feel more "at home" in the office.

(D). The advisor should try to minimize tension and stress by adjusting to meet the client's expectations and needs. The advisor should not provide less service or no service because of a difference in social styles. It is also unreasonable to hire another associate: there is a proper balance to be struck when accommodating clients.

A father wants to accumulate funds for his 12-year-old son's college education.On the advice of his attorney, the father establishes a Sec. 2503(c) trust and funds it with annual gifts. All of the following statements concerning this arrangement are correct EXCEPT: (A) The trust must be irrevocable. (B) In the event of the son's death prior to the age of majority, trust assets must either be payable to the son's estate or be subject to a general power of appointment held by the son. (C) Any accumulated income and all trust principal must be available for distribution to the son when he attains the age of majority. (D) The father's annual gift tax exclusion must be reduced by any amount used to pay college tuition costs.

(D). The annual gift tax exclusion is a separate exclusion from the educational and medical provider exclusions.

All of the following would be appropriate to assist in the financial planning process for blended families EXCEPT: (A) Review health insurance policies to ensure that all parties are properly insured. (B) Review life insurance policies to ensure that all beneficiaries are up to date and the death benefit amounts are still adequate. (C) Review property and casualty insurance policies to ensure that both spouses and all children/stepchildren are covered on the policies. (D) Review umbrella insurance policies to make sure coverage only extends to the biological parent.

(D). The first three statements are all types of financial planning that should be performed for blended families. Umbrella insurance policies should be reviewed as well, but the advisor should ensure that the umbrella coverage extends over both biological and nonbiological parents.

Which of the following statements identifies an approach that converts retirement assets into income that includes a focus on essential needs as part of the investment strategy? I. The minimum required distribution approach II. The systematic withdrawal approach (A) I only (B) II only (C) Both I and II (D) Neither I nor II

(D). The time-based segmentation approach (also known as the "bucket approach" or the "age-banded" approach) and the essential-versus-discretionary approach (also known as flooring) focus on investing for essential needs.

All of the following are approaches to convert retirement assets into retirement income EXCEPT: (A) The Systematic Withdrawal Approach (B) The Time-Based Segmentation Approach (also known as the "Bucket Approach" or the "Age-Banded" Approach) (C) The Essential-Versus-Discretionary Approach (also known as Flooring) (D) The Minimum Required Distribution Approach

(D). There is no strategy called the minimum required distribution retirement income strategy.

Hank and Luann are divorcing. How to split Luann's qualified retirement plan remains a contentious issue. All of the following options are available to them EXCEPT: (A) the buyout approach, through which Hank's share of the plan is 'bought out' with comparable assets (B) the wait-and-see approach, through which plan assets will be split at Luann's retirement (C) the reserved jurisdiction approach, through which assets are valued today but split in the future (D) the defined-contribution approach, through which the plan is split based on each party's proportional contribution

(D). There is no such thing as a defined-contribution approach to splitting qualified retirement plans at divorce. Defined-contribution plans are a type of retirement plan such as a 401(k) or a 403(b).

Each of the following are representative of behavioral finance biases, EXCEPT: (A) James saves $500 per month and has a purchase program to buy Amazon stock at average prices since he uses Amazon extensively and has a positive view of the company along with its products and services. (B) Tina is thrilled with her stock picking skills now that she has an account, which has appreciated 19% in one year. (C) Mary believes that she has a good grasp on the stock market and is proud that she was able to have foreseen the market crash of 2008. (D) Gordon maintains that his favorite stock should sell at 25 times forward earnings for this year, while his sister believes that it should sell at no more than a market multiple of earnings.

(D). This is an example of divergence of opinion. (A), (B), and (C) are examples of confirmation bias, overconfidence bias, and hindsight bias, respectively.

All of the following are market anomalies in behavioral finance EXCEPT: (A) low-volatility anomaly (B) value effect (C) small stock effect (D) individuals trade future consumption for current consumption

(D). This statement is supported by the neoclassical economic theory. (A), (B), (C) are anomalies discussed in behavioral finance literature.

Which of the following best demonstrates the interplay of life insurance planning for survivorship needs with the life-cycle planning approach? (A) Clients in pre-retirement and retirement are the least likely to need life insurance to provide financial resources for surviving children. (B) Because they are unlikely to pass away at a young age, clients in the career development and early career phases generally do not need life insurance. (C) Because they have relatively valuable assets, clients in the peak accumulation phase have the greatest life insurance need. (D) Clients in retirement have the greatest life insurance need because they are the most likely to pass away.

A. Life insurance is very often used to provide for surviving children in the event a client passes away before accumulating much wealth. As clients accumulate wealth and their children grow up and move out, their life insurance needs decline. In contrast, younger clients with young children have the greatest need for life insurance even though they are least likely to die. (C) is incorrect because clients with the most wealth can best self-insure against an early death (also, they generally have older children and therefore less survivorship need). Finally, (D) is incorrect because life insurance to cover survivorship needs is least appropriate for older clients even though they are the most likely to pass away.

"Past performance does not predict future results" is a statement that warns investors against which bias or heuristic? (A) Intertemporal inconsistencies (B) Availability biases (C) Gambler's Fallacy (D) Framing

B

Clearly defining your target market and product or service is an important component of: (A) communicating recommendations (B) client prospecting (C) defining the client/planner relationship (D) attaining credentials, such as the CFP® and ChFC

B. Successful prospecting means clearly defining your target market and the products and services you offer. (A) and (C) are incorrect because prospecting occurs before the start of the financial planning process that includes defining the client/planner relationship and communicating recommendations. Attaining credentials, choice (D), is also part of the prospecting process.

Each of the following statements regarding the two systems theory of behavioral finance are true EXCEPT: (A) The two systems theory of behavioral finance recognized that humans have two separate brain regions that are often simultaneously responsible for processing information. (B) Basic, faster, automatic responses that humans share with animals correspond to system 1. (C) Two systems theory also explains and merges with neoclassical theory. (D) Behavioral finance researchers often study the behavior of individual investors.

C

Which of the following statements support(s) the framing bias in decision making? I. Gain-framed decisions typically result in risk-avoidant preferences. II. Loss-framed decisions typically result in risk-seeking preferences. (A) I only (B) II only (C) Both I and II (D) Neither I nor II

C

A financial advisor receives a phone call from a client who wants help purchasing triple-leveraged stock index funds. Of the following options, which is the most appropriate next step for the advisor to take? (A) Execute an order for the fund with the lowest expense ratio. (B) Research alternative funds and compile a list. (C) Gather more data about the client, including investment goals and time-horizon. (D) Decline to work with this client, because triple-leveraged funds are too risky.

C. Before developing recommendations and alternatives (B) or implementing recommendations (A), an advisor must understand the client's goals by gathering data. Choice (D) is incorrect: without understanding the client's situation, it is impossible to know whether or not these funds are too risky or otherwise inappropriate for the client.

Which of the following correctly pairs a data gathering method with a benefit of that method? (A) Interviews help clients build a sense of ownership in the data gathering process. (B) Fact finders rely on clients to self-report. (C) Interviews are good for attaining qualitative information, such as goals. (D) Fact finders must be completed in the presence of the financial advisor.

C. Interviews—especially face-to-face interviews—are best for understanding clients' goals, needs, and other "soft" data. Fact finders, on the other hand, can be done without the financial advisor and may help clients build a sense of ownership in the process. Consequently, fact finders' reliance on client self-reporting may lead to errors or missing information.

A new financial advising firm is deciding on the most appropriate compensation model for their advisors. If they are trying to provide the most affordable service for their clients, they should adopt: (A) an Assets Under Management (AUM) model (B) a commission-based model (C) a retainer or subscription model (D) The answer depends on their clients' income, assets, goals, and needs.

D. No single compensation model is guaranteed to provide the most affordable service for clients. For example, AUM may be the most affordable for clients with few investable assets but the most expensive for clients with considerable assets (the opposite may be true for the retainer model). And while commissions are often singled out as problematic, they may reduce immediate out-of-pocket costs for some clients.

What is the number of years used as a rule of thumb to address a longevity risk? (A) 30 (B) 25 (C) 20 (D) 15

The answer is (A). Most studies on sustainable withdrawal rates use 30 years, which for retirement at age 65, tests to 95.

Five years before his death, Mr. Hope, the insured, transferred a life insurance policy on his life to his daughter but retained certain rights in the policy. Which of the following rights that Mr. Hope retained will cause the policy proceeds to be included in his gross estate for federal estate tax purposes? I. the right to surrender the policy but only with the consent of the daughter II. the right to receive policy premium notices (A) I only (B) II only (C) Both I and II (D) Neither I nor II

The answer is (A). The right to surrender the policy is an incident of ownership that will cause inclusion of the proceeds. It is irrelevant for Sec. 2042 purposes that Mr. Hope can only exercise this incident of ownership with the consent of another. Statement II is incorrect because the mere right to premium notices will not cause inclusion of policy proceeds in Mr. Hope's gross estate under Sec. 2042.

Which of the following is a task in Step 1 of the retirement income planning process? (A) Secure demographic, qualitative, and quantitative data from the client. (B) Secure historical information around the client's investment portfolio for the last 5 years. (C) Give the client a best estimate of what his/her retirement income needs may be. (D) Contact the client's other advisors, if any, including his/her accountant, attorney, and life insurance agent to confirm that the client has given correct information.

The answer is (A). This answer captures the purpose of Step 1; securing historical information is not needed, giving an estimate is not needed and contacting other advisors for this purpose would be beyond the scope of the mantra, "Trust, but verify," and could even cause fee and confidentiality issues.

Which of the following is true regarding long-term capital gains? (A) Most distributions from a 401(k) are stocks and are taxed at the individual's capital-gains tax rate. (B) Long-term capital-gains tax rates can be up to 28% but are often 15%. (C) If you are subject to the net investment income tax, all capital gains are taxed at your ordinary income tax rate. (D) Retirees often try to avoid long-term capital-gains tax rates because they are usually higher than their highest marginal federal income tax rates.

The answer is (B). Stocks and other capital assets are taxed at a special rate if held for a long enough period of time, which is 12 months or longer. The highest the rate can be is 28% for collectibles and 20% for stocks, but most people tend to see 15% or even lower for their long-term capital-gain tax rate. (A) is wrong because most distributions from a 401(k) are taxed as ordinary income. (C) is incorrect because the net investment income tax is just a new tax and does not directly impact your capital-gains tax rates. (D) is incorrect because long-term capital-gains tax rates tend to be lower rates than your ordinary income tax rates.

Which of the following statements best defines decumulation phase risk tolerance? (A) Decumulation phase risk tolerance is a synonym of investment risk tolerance. (B) Decumulation phase risk tolerance addresses a client's feeling on how concerned she is that her retirement income plan will use her assets at a rate that would cause the exhaustion of retirement income sources prior to the end of her life. (C) Decumulation risk tolerance encompasses a client's concerns about investment risk. (D) Decumulation risk tolerance is a concern only at the onset of retirement.

The answer is (B). This answer correctly defines the meaning of decumulation risk tolerance for the purposes of the retirement income planning process.

Donor establishes and funds an irrevocable trust for the benefit of Abe, Ben,and Kate. Income is to be paid for 15 years to either Abe or Ben, as directed by Donor. The remainder goes to Kate or Kate's estate. Which of the following statements concerning completed gifts for federal gift tax purposes is (are)correct? I. The remainder interest to Kate is a completed gift. II. The income interests designated in the trust terms to Abe and Ben are incomplete gifts. (A) I only (B) II only (C) Both I and II (D) Neither I nor II

The answer is (C). Statement I is correct because Donor retained no rights that would affect the remainder interest to Kate. Statement II is correct because Donor has the right to control and affect the income interests of Abe and Ben, and it is, therefore, an incomplete gift. However, as income is actually paid out to Abe and Ben, the amount paid constitutes a completed gift of a present interest.

Which of the following would not be subject to liquidity risk? (A) Real Estate (B) Closely Held Small Business (C) Microsoft Stock (D) Single-Premium Immediate Life Annuity

The answer is (C). Stocks do not have a lot of liquidity risk as they can be sold quickly. A specific stock like Microsoft might be subject to a variety of other risks, such as market- and company-specific risks, but not liquidity risk. (A) is wrong because real estate can be very difficult to sell quickly even if a buyer is found almost immediately. (B) is incorrect because closely held small businesses might not have a market. (D) is incorrect because life annuities have traditionally offered no liquidity.

Which of the following periods would best describe the time period for a client when he or she is suffering from some physical restrictions and permeating health limitations? (A) The "Final Act" period (B) Go-Go Years (C) Slow-Go Years (D) No-Go Years

The answer is (C). The Slow-Go years are often marked by health and other restrictions that limit activities. The Go-Go years are the early retirement years when the client is not burdened with health or other limitations. The No-Go years are the end-of-life years when health or other burdens severely limit the client's activities. The "Final Act" period can refer to the end-of-life years or the planning stage right after death, including estate planning.

Which of the following is true about Social Security retirement benefits? (A) Social Security retirement benefits are calculated based on your highest 40 years of work history. (B) For years in which you do not work, the taxable wage base is substituted in as the default amount for your benefit calculations. (C) All Americans are entitled to a nominal amount of Social Security retirement benefits. (D) Collecting Social Security benefits before your full retirement age will result in a permanent reduction of benefits.

The answer is (D). Persons can retire as early as age 62, but the monthly benefit is subject to an early retirement reduction. The reduction is 5/9 of 1 percent for each of the first 36 months of entitlement immediately preceding the age at which 100 percent of PIA is payable (scheduled to increase to age 67 by the year 2022),plus 5/12 of 1 percent for each of up to 24 earlier months. (A) is incorrect because Social Security benefits are calculated based on 35years of wage history. (B) is wrong because in years you do not work, $0s are placed in as your wages. (C) is wrong because there is no amount that everyone is entitled to under Social Security.

Which of the following statements concerning the filing of the federal estate tax return is (are) correct? I. An estate tax return must be filed for all U.S. citizens dying after 2001. II. There is an automatic 2-year extension for filing the federal estate tax return of a U.S. citizen who dies in a foreign country. (A) I only (B) II only (C) Both I and II (D) Neither I nor II

The answer is (D). Statement I is incorrect because at the present time, filing an estate tax return is required only when the gross estate plus adjusted taxable gifts exceeds the basic exclusion/credit amount. Statement II is incorrect because filing extensions are not automatic. Generally, an extension to file must be received and granted by the IRS before the time for filing the estate tax return expires.

Which of the following powers held by a trust beneficiary will cause all the assets in the trust to be included in the beneficiary's gross estate for federal estate tax purposes? I. the power to direct the trustee to pay her the greater of 5 percent of the trust principal or $5,000 II. the power to use trust assets for her health, education, maintenance, or support (A) I only (B) II only (C) Both I and II (D) Neither I nor II

The answer is (D). The IRS considers both powers to be de minimus, limited, and ascertainable powers that do not cause inclusion of the value of the trust in the powerholder's estate.


संबंधित स्टडी सेट्स

ATI RN Mental Health Nursing Modules Ch. 10 Notes

View Set

PEDS: Ch. 16 Nursing Care of the Child With an Alteration in Intracranial Regulation/Neurologic Disorder

View Set

Ch 27: Seedless Plants-questions

View Set

Chapter 23: The Great Depression

View Set

WGU C963 American politics and US Constitution

View Set

Ch. 4 - Theories of Cognitive Development

View Set

Regular and Irregular Plural Nouns

View Set

Chapter 8: Communication Climate

View Set

Economics - Market Equilibrium and Policy

View Set